Difference between revisions of "2021 Fall AMC 12B Problems/Problem 25"
(→Solution 4) |
(→Solution 4) |
||
(4 intermediate revisions by the same user not shown) | |||
Line 159: | Line 159: | ||
==Solution 4== | ==Solution 4== | ||
− | Upon adding one to <math>n</math>, consider each individual remainder. Either it will increase by 1, or it will "wrap around', going from <math>5\rightarrow 0</math> mod 6 and in general, <math>(n-1) \rightarrow 0</math> mod n. We will use '<math>+1</math>' to refer to remainders that increase by 1, and 'wrap-around's to refer to remainders that go to 0. | + | Upon adding one to <math>n</math>, consider each individual remainder. Either it will increase by 1, or it will "wrap around', going from <math>5\rightarrow 0</math> (mod 6) and in general, <math>(n-1) \rightarrow 0</math> (mod n). We will use '<math>+1</math>' to refer to remainders that increase by 1, and 'wrap-around's to refer to remainders that go to 0. |
Line 165: | Line 165: | ||
− | If there are <math>8</math> <math>+1</math>s and <math>1</math> wrap-around, the wrap-around must be equal to <math>-8</math>, which is the case for <math>(9)</math>. However, if <math>n</math> is <math>8</math> mod <math>9</math>, it clearly must also be <math>2</math> mod <math>3</math>, meaning | + | If there are <math>8</math> <math>+1</math>s and <math>1</math> wrap-around, the wrap-around must be equal to <math>-8</math>, which is the case for mod <math>(9)</math>. However, if <math>n</math> is <math>8</math> mod <math>9</math>, it clearly must also be <math>2</math> mod <math>3</math>, meaning mod (3) must also be a wrap-around, and this case won't work. |
Line 174: | Line 174: | ||
− | Notice that there can be no more cases, as for <math>5</math> <math>+1</math>s, no matter what mods wrap around, the <math>+1</math>s will not be able to balance them out, as it's magnitude is too small. Therefore, there are only <math>\boxed{\textbf{C) }2}</math> numbers. | + | Notice that there can be no more cases, as for <math>5</math> <math>+1</math>s, no matter what mods wrap around, the <math>+1</math>s will not be able to balance them out, as it's magnitude is too small. Therefore, there are only <math>\boxed{\textbf{C) }2}</math> numbers, namely <math>13</math> and <math>97</math>. |
+ | |||
+ | ~skibbysiggy | ||
==Video Solution== | ==Video Solution== |
Latest revision as of 19:34, 4 November 2024
Contents
Problem
For a positive integer, let
be the sum of the remainders when
is divided by
,
,
,
,
,
,
,
, and
. For example,
. How many two-digit positive integers
satisfy
Solution 1
Note that we can add to
to get
, but must subtract
for all
. Hence, we see that there are four ways to do that because
. Note that only
is a plausible option, since
indicates
is divisible by
,
indicates that
is divisible by
,
indicates
is divisible by
, and
itself indicates divisibility by
, too. So,
and
is not divisible by any positive integers from
to
, inclusive, except
and
. We check and get that only
and
give possible solutions so our answer is
.
- kevinmathz
Solution 2
Denote by the remainder of
divided by
.
Define
.
Hence,
Hence, this problem asks us to find all , such that
.
:
.
We have .
Therefore, there is no in this case.
:
and
.
The condition implies
.
This further implies
.
Hence,
.
To get , we have
.
However, we have .
Therefore, there is no in this case.
:
for
and
.
The condition implies
with
.
Hence,
and
.
To get , we have
.
However, we have .
Therefore, there is no in this case.
:
for
and
.
To get , we have
.
Hence, we must have and
for
.
Therefore, .
:
for
and
.
The condition implies
with
.
Hence,
and
.
To get , we have
.
However, we have .
Therefore, there is no in this case.
:
for
and
.
To get , we have
.
This can be achieved if ,
,
.
However, implies
. This implies
. Hence,
.
We get a contradiction.
Therefore, there is no in this case.
:
for
and
.
The condition implies
with
.
Hence,
.
To get , we have
. This implies
.
Because and
, we have
.
Hence,
.
However, in this case, we assume
.
We get a contradiction.
Therefore, there is no in this case.
:
for
and
.
To get , we have
. This is infeasible.
Therefore, there is no in this case.
:
for
.
To get , we have
. This is infeasible.
Therefore, there is no in this case.
Putting all cases together, the answer is .
~Steven Chen (www.professorchenedu.com)
Solution 3
To get from to
,
would add by
for each remainder
. However, given that some of these remainders can "round down" to
given the nature of mods, we must calculate the possible values of
such that the remainders in
"rounds down" by a total of
, effectively canceling out the adding by
initially.
To do so, we will analyze the "rounding down" for each of :
: subtract by
: subtract by
: subtract by
, but this also implies mod
, so subtract by
.
: subtract by
: subtract by
, but this also implies mod
and
, so subtract by
: too much
: subtract by
: subtract by
, but this also implies mod
and
, so subtract by
: too much
: subtract by
, but this also implies mod
, so subtract by
: too much
: subtract by
: too much
Notice that . By testing these sums, we can easily show that the only time when the total subtraction is
is when
AND
. By CRT,
:
As in solution 1, then, only and
give possible solutions, so our answer is
.
~xHypotenuse
Solution 4
Upon adding one to , consider each individual remainder. Either it will increase by 1, or it will "wrap around', going from
(mod 6) and in general,
(mod n). We will use '
' to refer to remainders that increase by 1, and 'wrap-around's to refer to remainders that go to 0.
Clearly,
s isn't possible, since then
.
If there are
s and
wrap-around, the wrap-around must be equal to
, which is the case for mod
. However, if
is
mod
, it clearly must also be
mod
, meaning mod (3) must also be a wrap-around, and this case won't work.
If there are
s and
wrap-arounds, these two wrap-arounds must add to
. For the possible modulo, we could have
,
, and
. Clearly,
won't work since if it is
mod
, then it must also be
mod
, meaning
won't be the only wrap-arounds. Similarly,
doesn't work since
mod
implies that
mod
will also be a wrap-around. That leaves
. The number must be
mod
and
mod
, or in other words,
mod
and
mod
, meaning n will be
mod
. Testing all such two digits numbers that are equivalent to
mod
, we see that
and
are the only two that work.
If there are
s and
wrap-arounds, the only possible combination of modulo is
. Thus,
must be
mod
. However, this means that mod
will also be a wrap around, so this case won't work.
Notice that there can be no more cases, as for
s, no matter what mods wrap around, the
s will not be able to balance them out, as it's magnitude is too small. Therefore, there are only
numbers, namely
and
.
~skibbysiggy
Video Solution
~MathProblemSolvingSkills.com
Video Solution by Mathematical Dexterity
https://www.youtube.com/watch?v=Fy8wU4VAzkQ
2021 Fall AMC 12B (Problems • Answer Key • Resources) | |
Preceded by Problem 24 |
Followed by Last problem |
1 • 2 • 3 • 4 • 5 • 6 • 7 • 8 • 9 • 10 • 11 • 12 • 13 • 14 • 15 • 16 • 17 • 18 • 19 • 20 • 21 • 22 • 23 • 24 • 25 | |
All AMC 12 Problems and Solutions |
The problems on this page are copyrighted by the Mathematical Association of America's American Mathematics Competitions.